LSAT and Law School Admissions Forum

Get expert LSAT preparation and law school admissions advice from PowerScore Test Preparation.

User avatar
 Dave Killoran
PowerScore Staff
  • PowerScore Staff
  • Posts: 5853
  • Joined: Mar 25, 2011
|
#41280
Complete Question Explanation
(The complete setup for this game can be found here: lsat/viewtopic.php?t=11660)

The correct answer choice is (E)

Since this question focuses specifically on days 3 and 5, again apply the Not Laws from day 3 and day 5. There are none for day 3, but we see that J cannot be inspected on day 5, and so we can kill off answer choice (D), which attempts to place J in day 5. Now, apply linkage to help eliminate more answer choices: Day 3 appears in the question and it also appears in the last rule. Thus, as you should recall, if G is inspected on day 3, then Q must be inspected on day 5. This information is sufficient to eliminate answer choices (A) and (B) since both have G on day 3 but another variable besides Q on day 5. Now we are down to only two remaining answer choices, yet we have only had to do a minimal amount of work. At this point, quickly scan the two remaining answer choices to see if you can identify the correct answer without doing further work. If you cannot, why not use one of the answer choices to help make a hypothetical that will solve the problem? To prove a “Could Be True” answer, all that is needed is one hypothetical that shows that one of the scenarios is possible, or alternately, a hypothetical that shows that one of the scenarios is impossible. Let us try answer choice (C) first. Make the following notation right beside the problem:

J96_Game_#1_#4_diagram 1.png
Diagramming note: We did not write out the numbers since most people do not need the numbers when they work next to the question. Had this been the main diagram you would most certainly have wanted to write out the number for each space.

As you might see, this scenario presents a problem because there is no room for the QR block. If F is inspected on day 1, there are not two consecutive spaces available for the QR block. If F is inspected on day 6, the only two spaces available for Q and R are days 1 and 2, but that leaves no room for J and so answer choice (C) cannot be correct. By process of elimination this means answer choice (E) must be correct. Notice that we do not need to prove that (E) is possible since we have already eliminated each of the other answer choices. Since the other four are incorrect, answer choice (E) must be correct. Mark it and continue. For those of you wondering if answer choice (E) can fit in a valid solution, here is a hypothetical that fits (E) and fits all of the rules of the game: J-Q-R-G-H-F.
You do not have the required permissions to view the files attached to this post.

Get the most out of your LSAT Prep Plus subscription.

Analyze and track your performance with our Testing and Analytics Package.